Does the conclusion escape you? Has understanding the tone of the passage gotten you down? Get help here.
Tadashi
Forum Guests
 
Posts: 126
Joined: Mon Apr 07, 2014 5:02 pm
 

Re: An economic recession can result from a lowering

by Tadashi Mon May 26, 2014 2:49 am

Hi Ron,
I googled the stem of this SC problem and found this thread.
Therefore, I think I don't have to open a new thread.

My question about A.
1. Is "a lowering of employment rates" not ideal? I think "a gerund" of some noun. is quite weird.. sounds awkward.

2. Choice A seems to convey the idea that a drop in investment causes three parallel things: falling employment rates, cutbacks in consumer spending, and a cycle of layoffs.



My question about C.(OA)
1. "that lead to even lower employment rates." modifies "cutbacks in consumer spending and starting a cycle of layoffs" as a whole.

Please make comments on my thoughts.
ARIGATO
Tadashi.
RonPurewal
Students
 
Posts: 19744
Joined: Tue Aug 14, 2007 8:23 am
 

Re: An economic recession can result from a lowering

by RonPurewal Thu Jun 05, 2014 5:11 am

Hmm. I don't like this problem. I'm going to submit it for editing.

- "A lowering" is not awkward, but it's inappropriate here. "A lowering" seems to imply that someone lowered something on purpose.

- "Falling employment rates" would be fine by itself"”but "falling employment rates triggered by X" is illogical. The rates aren't triggered by xxxx"”the fall is!

The correct answer should say "a decrease in...", or "a fall in...", or something similar.

Also, readers without a knowledge of economics won't be able to determine exactly what the modifier ("causing cutbacks...") should modify.
The GMAT doesn't require any specialized knowledge, so that's not good, either.
RonPurewal
Students
 
Posts: 19744
Joined: Tue Aug 14, 2007 8:23 am
 

Re: An economic recession can result from a lowering

by RonPurewal Thu Jun 05, 2014 5:13 am

This question is not active in the CAT database. Where did you see it?

Thanks.
Tadashi
Forum Guests
 
Posts: 126
Joined: Mon Apr 07, 2014 5:02 pm
 

Re: An economic recession can result from a lowering

by Tadashi Fri Jun 06, 2014 1:30 am

in the Manhattan non-CAT sc question bank I think. [i bought both CAT and non-CAT problem set. ]
maybe I should check it. i can't remember it clearly. :(
sorry.
RonPurewal
Students
 
Posts: 19744
Joined: Tue Aug 14, 2007 8:23 am
 

Re: An economic recession can result from a lowering

by RonPurewal Thu Jun 12, 2014 4:11 am

Tadashi Wrote:in the Manhattan non-CAT sc question bank I think. [i bought both CAT and non-CAT problem set. ]
maybe I should check it. i can't remember it clearly. :(
sorry.


Please do. Thanks.
study_gmat
Course Students
 
Posts: 11
Joined: Wed Jan 11, 2012 12:48 pm
 

Re: An economic recession can result from a lowering

by study_gmat Sat Jun 21, 2014 8:45 pm

Hello,

I just ran across this problem in the SC drill set as well.

My question is regarding why the "which" can't modify "drop in investment" since this is what causes people to send less? In the correct answer is "causing" is still referring to a "drop in investment"?

Thanks
RonPurewal
Students
 
Posts: 19744
Joined: Tue Aug 14, 2007 8:23 am
 

Re: An economic recession can result from a lowering

by RonPurewal Wed Jun 25, 2014 7:46 am

study_gmat Wrote:Hello,

I just ran across this problem in the SC drill set as well.

My question is regarding why the "which" can't modify "drop in investment" since this is what causes people to send less?


The point of the sentence seems to be that, when people lose their jobs, they don't spend as much money.
Hence, falling employment rates "”> less spending.

On the other hand, there is absolutely no way you'll ever have to think in such specialized terms for a GMAT SC sentence. For any SC sentence on the official exam, you should be able to determine the intended meaning with common sense alone.
lsyang1212
Course Students
 
Posts: 19
Joined: Wed Aug 10, 2011 9:49 am
 

Re: An economic recession can result from a lowering

by lsyang1212 Wed Jul 02, 2014 3:28 pm

An economic recession can result from a lowering of employment rates triggered by a drop in investment, which causes people to cut consumer spending and starts a cycle of layoffs leading back to even lower employment rates.

OA: C: falling employment rates triggered by a drop in investment, causing cutbacks in consumer spending and starting a cycle of layoffs that lead to even lower employment rates.

Could you clarify what noun is "causing cutbacks in consumer spending..."? Is it the falling employment rates? The falling employment rates triggered by a drop in investment? The economic recession?

I'm having a hard time figuring out what an '-ing' clause is modifying. These are verb/phrase modifiers, not noun modifiers right? So they are used more losely. Not sure what the rule of thumb is.


Thanks.
RonPurewal
Students
 
Posts: 19744
Joined: Tue Aug 14, 2007 8:23 am
 

Re: An economic recession can result from a lowering

by RonPurewal Thu Jul 03, 2014 8:26 am

lsyang1212, your question appears to be answered by the post directly above.
RitikK630
Students
 
Posts: 5
Joined: Wed Jun 17, 2015 1:17 am
 

Re: An economic recession can result from a lowering

by RitikK630 Tue Jul 19, 2016 1:38 pm

Hi Ron,

Apologies for reopening an old thread. I am very confused with the usage of 'which' modifier and need your inputs on the below statement.

An economic recession can result from falling employment rates triggered by a drop in investment, which cause cutbacks in consumer spending, starting a cycle of layoffs that lead to even lower employment rates.

In the above mentioned sentence what does the modifier 'which' refer to. Does it refer to 'investment' or to 'drop in investment' or to 'rates'.
Also, how to identify which part the 'which' modifier is referring to ?

Is there any general rule that we can follow regarding the usage of 'which' for these type of sentences?
cgentry
ManhattanGMAT Staff
 
Posts: 44
Joined: Wed Aug 04, 2010 3:28 am
 

Re: An economic recession can result from a lowering

by cgentry Thu Oct 06, 2016 3:55 pm

RitikK630 Wrote:
An economic recession can result from falling employment rates triggered by a drop in investment, which cause cutbacks in consumer spending, starting a cycle of layoffs that lead to even lower employment rates.

In the above mentioned sentence what does the modifier 'which' refer to. Does it refer to 'investment' or to 'drop in investment' or to 'rates'.
Also, how to identify which part the 'which' modifier is referring to ?

Is there any general rule that we can follow regarding the usage of 'which' for these type of sentences?


In your example, the ", which" must describe "falling employment rates". (I've kept the essential modifiers "falling" and "employment" for meaning; "rates" is the only noun in that phrase.)

A ", which" modifier is a clause modifier: there is a verb inside the modifier itself. Because of this, there are two issues we must consider--placement and subject/verb (S/V) agreement.

In your example, placement would indicate that the modifier describes "investment". But investment is singular, and "cause"--the verb inside the modifier--is plural. This singular vs plural disagreement means the modifier cannot describe investment. We must find a plural noun preceding the modifier, and the only one is "rates".

If you see a ", which" (or ",where" or ", who") modifier, consider both placement and S/V agreement between the verb in the modifier and the noun you are attempting to describe. If both placement and verb agree, then you have the noun that is described. For example, consider the sentence "I teach classes at the Georgia Tech Hotel and Conference Center, which is located in midtown." Here, the placement indicates that the modifier is describing the GT Hotel and Conference Center, and since both this noun and the verb in the modifier ("is") are singular, this modifier is describing what it's touching.

In your example, since the placement and S/V relationship do not indicate the same noun. In this case, the S/V agreement takes precedence.
KashifK568
Students
 
Posts: 2
Joined: Fri May 26, 2017 6:16 pm
 

Re: An economic recession can result from a lowering

by KashifK568 Mon May 29, 2017 2:04 pm

Option D
falling employment rates that are triggered[/b] by a drop in investment, causing people to cut consumer spending and starting a cycle of layoffs that lead back to even lower employment rates.


dbernst Wrote:aaa, I tend to agree with Guest79 - the first "that" is awkward and unnecessary;

-dan


could someone elaborate on why it is awkward? After all, the "that" is used in the right answer choice.

layoffs that lead to even lower employment rates.
layoffs leading to even lower employment rates <-- we could use this, but then it implies "layoffs are doing the leading" so it may be wrong.

Can 'that' not modify 'falling employment rates'? What is the error here?
RonPurewal
Students
 
Posts: 19744
Joined: Tue Aug 14, 2007 8:23 am
 

Re: An economic recession can result from a lowering

by RonPurewal Wed May 31, 2017 3:14 pm

this question was deactivated at least a couple years ago, so, i'm not sure how people are even reaching this page. (are there collections of our old exam questions floating around the internet?)

in any case, i'm going to lock this thread, since the discussion is no longer relevant.

__

in response to this most recent question—"awkward" isn't a thing that has reasons.

also, "awkward" is ...
...clearly not tested on this exam,
...essentially impossible to judge if you haven't been familiar with a language since childhood. (not just english, but, basically any language)

if you see "awkward" in OG answer keys, that basically means the answer-key writer didn't understand how to accurately describe what's really wrong with that choice. (the OG answer keys were clearly written by people far, far less talented than the actual question writers.)